1. Joined
    29 Feb '04
    Moves
    22
    12 Mar '05 15:213 edits
    On an empty chessboard
    let the total number of moves a King can make = K
    let the total number of moves a Queen can make = Q
    let the total number of moves a Rook can make = R
    let the total number of moves a Bishop can make = B
    let the total number of moves a Knight can make = N
    let the total number of moves a Pawn can make = P

    eg. R = (64)(14) = 896
    because, wherever you place the Rook, it always attacks 14 squares.

    1a) What is the relationship between R, B, and N? Is this a coincidence?

    1b) Are there other simple relationships between any of the six pieces?

    On an nxn chessboard
    let the total number of moves a King can make = K(n) for n > 0
    let the total number of moves a Queen can make = Q(n) for n > 0
    let the total number of moves a Rook can make = R(n) for n > 0
    let the total number of moves a Bishop can make = B(n) for n > 0
    let the total number of moves a Knight can make = N(n) for n > 1
    let the total number of moves a Pawn can make = P(n) for n > 3

    2) Find K(n), Q(n), R(n), B(n), N(n), P(n)

    On an nxm chessboard
    let the total number of moves a King can make = K(n,m) for n,m > 0
    let the total number of moves a Queen can make = Q(n,m) for n > m
    let the total number of moves a Rook can make = R(n,m) for n,m > 0
    let the total number of moves a Bishop can make = B(n,m) for n > m
    let the total number of moves a Knight can make = N(n,m) for n,m > 1

    3) Find K(n,m), Q(n,m), R(n,m), B(n,m), N(n,m)

    Let the total number of moves a Pawn can make on an rxf board = P(r,f) for r > 3, f > 0

    4) Find P(r,f)

    5) Prove that there is only a finite number of boards such that R(i,j) = B(i,j) + N(i,j)


    6) For what size boards are there such simple relationships between the pieces?

    Define a Jumper as a Knight which can jump pxq (p =< q) rather than only 1x2.
    Let the total number of moves a Jumper can make = J(n,m,p,q) for n,m >= q)

    7) Find J(n,m,p,q) when (i) p = 0 (ii) p = q (iii) 0 < p < q

    .
  2. Joined
    04 Aug '01
    Moves
    2408
    13 Apr '05 22:24
    These are good questions...I am trying to chip them away slowly.

    For 1A:

    By my calculations I get B = 280 and N = 336, so together they satisfy

    2B + N = R.

    Not a coincidence.
  3. Joined
    29 Feb '04
    Moves
    22
    14 Apr '05 02:09
    Nope, your Bishop formula must be wrong.

    By the way, P(n,m) includes captures and en passant.
  4. Joined
    04 Aug '01
    Moves
    2408
    14 Apr '05 03:03
    Originally posted by THUDandBLUNDER
    Nope, your Bishop formula must be wrong.

    By the way, P(n,m) includes captures and en passant.
    Hmmm...maybe i misunderstood initially. By B do you mean the number of moves a bishop can make, assuming he is not confined to one color (I was assuming one color only)?

    In that case it would be 2 X 280 = 560, and then

    B + N = R.

    Still not a coincidence.

    If that's not right, then I think I am missing something...
  5. Joined
    29 Feb '04
    Moves
    22
    14 Apr '05 03:171 edit
    Originally posted by davegage
    If that's not right, then I think I am missing something...
    Yeah, that's right.
    Why do you think it is not a coincidence?
  6. Joined
    04 Aug '01
    Moves
    2408
    14 Apr '05 04:22
    Originally posted by THUDandBLUNDER

    Why do you think it is not a coincidence?
    Not sure yet. I was hoping that if I stated it matter-of-factly you would just buy that I knew what I was talking about...my plan seems to have failed...

    it seems a reasonable relationship, but still not quite sure how to rationalize it yet...
Back to Top

Cookies help us deliver our Services. By using our Services or clicking I agree, you agree to our use of cookies. Learn More.I Agree